Приемы решения иррациональных уравнений и неравенств

Алгебра

План урока:

Иррациональные уравнения

Ранее мы рассматривали целые и дробно-рациональные уравнения. В них выражение с переменной НЕ могло находиться под знаком радикала, а также возводиться в дробную степень. Если же переменная оказывается под радикалом, то получается иррациональное уравнение.

Приведем примеры иррациональных ур-ний:

Заметим, что не всякое уравнение, содержащее радикалы, является иррациональным. В качестве примера можно привести

Это не иррациональное, а всего лишь квадратное ур-ние. Дело в том, что под знаком радикала стоит только число 5, а переменных там нет.

Простейшие иррациональные уравнения

Начнем рассматривать способы решения иррациональных уравнений. В простейшем случае в нем справа записано число, а вся левая часть находится под знаком радикала. Выглядит подобное ур-ние так:

где а – некоторое число (константа), f(x) – рациональное выражение.

Для его решения необходимо обе части возвести в степень n, тогда корень исчезнет:

Получаем рациональное ур-ние, решать которые мы уже умеем. Однако есть важное ограничение. Мы помним, что корень четной степени всегда равен положительному числу, и его нельзя извлекать из отрицательного числа. Поэтому, если в ур-нии

n – четное число, то необходимо, чтобы а было положительным. Если же оно отрицательное, то ур-ние не имеет корней. Но на нечетные n такое ограничение не распространяется.

Пример. Решите ур-ние

Решение. Справа стоит отрицательное число (– 6), но квадратный корень (если быть точными, то арифметический квадратный корень) не может быть отрицательным. Поэтому ур-ние корней не имеет.

Ответ: корней нет.

Пример. Решите ур-ние

Решение. Теперь справа стоит положительное число, значит, мы имеем право возвести обе части в квадрат. При этом корень слева исчезнет:

Пример. Решите ур-ние

Решение. Справа стоит отрицательное число, но это не является проблемой, ведь кубический корень может быть отрицательным. Возведем обе части в куб:

Конечно, под знаком корня может стоять и более сложное выражение, чем (х – 5).

Пример. Найдите решение ур-ния

Решение. Возведем обе части в пятую степень:

х 2 – 14х – 32 = 0

Получили квадратное ур-ние, которое можно решить с помощью дискриминанта:

D = b 2 – 4ac = (– 14) 2 – 4•1•(– 32) = 196 + 128 = 324

Итак, нашли два корня: (– 2) и 16.

Несколько более сложным является случай, когда справа стоит не постоянное число, а какое-то выражение с переменной g(x). Алгоритм решения тот же самый – необходимо возвести в степень ур-ние, чтобы избавиться от корня. Но, если степень корня четная, то необходимо проверить, что полученные корни ур-ния не обращают правую часть, то есть g(x), в отрицательное число. В противном случае их надо отбросить как посторонние корни.

Пример. Решите ур-ние

Решение. Возводим обе части во вторую степень:

х – 2 = х 2 – 8х + 16

D = b 2 – 4ac = (– 9) 2 – 4•1•18 = 81 – 72 = 9

Получили два корня, 3 и 6. Теперь проверим, во что они обращают правую часть исходного ур-ния (х – 4):

при х = 3 х – 4 = 3 – 4 = – 1

при х = 6 6 – 4 = 6 – 4 = 2

Корень х = 3 придется отбросить, так как он обратил правую часть в отрицательное число. В результате остается только х = 6.

Пример. Решите ур-ние

Решение. Здесь используется кубический корень, а потому возведем обе части в куб:

3х 2 + 6х – 25 = (1 – х) 3

3х 2 + 6х – 25 = 1 – 3х + 3х 2 – х 3

Получили кубическое ур-ние. Решить его можно методом подбора корня. Из всех делителей свободного коэффициента (– 26) только двойка обращает ур-ние в верное равенство:

Других корней нет. Это следует из того факта, что функция у = х 3 + 9х – 26 является монотонной.

Заметим, что если подставить х = 2 в левую часть исходного ур-ния 1 – х, то получится отрицательное число:

при х = 2 1 – х = 1 – 2 = – 1

Но означает ли это, что число 2 НЕ является корнем? Нет, ведь кубический корень вполне может быть и отрицательным (в отличие от квадратного). На всякий случай убедимся, что двойка – это действительно корень исходного уравнения:

Уравнения с двумя квадратными корнями

Ситуация осложняется, если в ур-нии есть сразу два квадратных корня. В этом случае их приходится убирать последовательно. Сначала мы переносим слагаемые через знак «=» таким образом, чтобы слева остался один из радикалов и ничего, кроме него. Возводя в квадрат такое ур-ние, мы избавимся от одного радикала, после чего мы получим более простое ур-ние. После получения всех корней надо проверить, какие из них являются посторонними. Для этого их надо просто подставить в исходное ур-ние.

Пример. Решите ур-ние

Решение. Перенесем вправо один из корней:

Возведем обе части в квадрат. Обратите внимание, что левый корень при этом исчезнет, а правый – сохранится:

Теперь снова перемещаем слагаемые так, чтобы в одной из частей не осталось ничего, кроме корня:

Снова возведем ур-ние в квадрат, чтобы избавиться и от второго корня:

(2х – 4) 2 = 13 – 3х

4х 2 – 16х + 16 = 13 – 3х

4х 2 – 13х + 3 = 0

D = b 2 – 4ac = (– 13) 2 – 4•4•3 = 169 –48 = 121

Имеем два корня: 3 и 0,25. Но вдруг среди них есть посторонние? Для проверки подставим их в исходное ур-ние. При х = 0,25 имеем:

Получилось ошибочное равенство, а это значит, что 0,25 не является корнем ур-ния. Далее проверим х = 3

На этот раз получилось справедливое равенство. Значит, тройка является корнем ур-ния.

Введение новых переменных

Предложенный метод последовательного исключения радикалов плохо работает в том случае, если корни не квадратные, а имеют другую степень. Рассмотрим ур-ние

Последовательно исключить корни, как в предыдущем примере, здесь не получится (попробуйте это сделать самостоятельно). Однако помочь может замена переменной.

Для начала перепишем ур-ние в более удобной форме, когда вместо корней используются степени:

х 1/2 – 10х 1/4 + 9 = 0

Теперь введем переменную t = x 1/4 . Тогда х 1/2 = (х 1/4 ) 2 = t 2 . Исходное ур-ние примет вид

Это квадратное ур-ние. Найдем его корни:

D = b 2 – 4ac = (– 10) 2 – 4•1•9 = 100 – 36 = 64

Получили два значения t. Произведем обратную замену:

х 1/4 = 1 или х 1/4 = 9

Возведем оба ур-ния в четвертую степень:

(х 1/4 ) 4 = 1 4 или (х 1/4 ) 4 = 3 4

х = 1 или х = 6561

Полученные числа необходимо подставить в исходное ур-ние и убедиться, что они не являются посторонними корнями:

В обоих случаях мы получили верное равенство 0 = 0, а потому оба числа, 1 и 6561, являются корнями ур-ния.

Пример. Решите ур-ние

х 1/3 + 5х 1/6 – 24 = 0

Решение. Произведем замену t = x 1/6 , тогда х 1/3 = (х 1/6 ) 2 = t 2 . Исходное ур-ние примет вид:

Его корни вычислим через дискриминант:

D = b 2 – 4ac = 5 2 – 4•1•(– 24) = 25 + 96 = 121

Далее проводим обратную заменуx 1/6 = t:

х 1/6 = – 8 или х 1/6 = 3

Первое ур-ние решений не имеет, а единственным решением второго ур-ния является х = 3 6 = 729. Если подставить это число в исходное ур-ние, то можно убедиться, что это не посторонний корень.

Замена иррационального уравнения системой

Иногда для избавления от радикалов можно вместо них ввести дополнительные переменные и вместо одного иррационального ур-ния получить сразу несколько целых, которые образуют систему. Это один из самых эффективных методов решения иррациональных уравнений.

Пример. Решите ур-ние

Решение. Заменим первый корень буквой u, а второй – буквой v:

Исходное ур-ние примет вид

Если возвести (1) и (2) в куб и квадрат соответственно (чтобы избавиться от корней), то получим:

Ур-ния (3), (4) и (5) образуют систему с тремя неизвестными, в которой уже нет радикалов:

Попытаемся ее решить. Сначала сложим (4) и (5), ведь это позволит избавиться от переменной х:

(х + 6) + (11 – х) = u 3 + v 2

из (3) можно получить, что v = 5 – u. Подставим это в (6) вместо v:

17 = u 3 + (5 – u) 2

17 = u 3 + u 2 – 10u + 25

u 3 + u 2 – 10u + 8 = 0

Получили кубическое ур-ние. Мы уже умеем решать их, подбирая корни. Не вдаваясь в подробности решения, укажем, что корнями этого ур-ния являются числа

подставим полученные значения в (4):

x + 6 = 1 3 или х + 6 = 2 3 или х + 6 = (– 4) 3

x + 6 = 1 или х + 6 = 8 или х + 6 = – 64

х = – 5 или х = 2 или х = – 70

Итак, нашли три возможных значения х. Но, конечно же, среди них могут оказаться посторонние корни. Поэтому нужна проверка – подставим полученные результаты в исходное ур-ние. При х = – 5 получим

Корень подошел. Проверяем следующее число, х = 2:

Корень снова оказался верным. Осталась последняя проверка, для х = – 70:

Итак, все три числа прошли проверку.

Уравнения с «вложенными» радикалами

Порою в ур-нии под знаком радикала стоит ещё один радикал. В качестве примера приведем такую задачу:

При их решении следует сначала избавиться от «внешнего радикала», после чего можно будет заняться и внутренним. То есть в данном случае надо сначала возвести обе части равенства в квадрат:

Внешний радикал исчез. Теперь будем переносить слагаемые, чтобы в одной из частей остался только радикал:

Хочется поделить полученное ур-ние (1) на х, однако важно помнить, что деление на ноль запрещено. То есть, если мы делим на х, то мы должны наложить дополнительное ограничение х ≠ 0. Случай же, когда х всё же равен нулю, мы рассматриваем отдельно. Для этого подставим х = 0 сразу в исходное ур-ние:

Получили верное рав-во, значит, 0 является корнем. Теперь возвращаемся к (1) и делим его на х:

Возводим в квадрат и получаем:

х 2 + 40 = (х + 4) 2

х 2 + 40 = х 2 + 8х + 16

И снова нелишней будет проверка полученного корня:

Иррациональные неравенства

По аналогии с иррациональными ур-ниями иррациональными неравенствами называют такие нер-ва, в которых выражение с переменной находится под знаком радикала или возводится в дробную степень. Приведем примеры иррациональных нер-в:

Нет смысла решать иррациональные нер-ва, если есть проблемы с более простыми, то есть рациональными нер-вами, а также с их системами. Поэтому на всякий случай ещё раз просмотрите этот и ещё вот этот уроки.

Начнем с решения иррациональных неравенств простейшего вида, у которых в одной из частей стоит выражение под корнем, а в другой – постоянное число. Достаточно очевидно, что нер-во вида

Может быть справедливым только тогда, когда

То есть, грубо говоря, нер-ва можно возводить в степень. Однако при этом могут возникнуть посторонние решения. Дело в том, что нужно учитывать и тот факт, что подкоренное выражение должно быть неотрицательным в том случае, если степень корня является четной. Таким образом, нер-во

при четном n можно заменить системой нер-в

Пример. При каких значениях x справедливо нер-во

Решение. С одной стороны, при возведении нер-ва в квадрат мы получим такое нер-во:

х ⩽ – 5 (знак нер-ва изменился из-за того, что мы поделили его на отрицательное число)

Получили промежуток х∈(– ∞; – 5). Казалось бы, надо записать ещё одно нер-во

чтобы подкоренное выражение было неотрицательным. Однако сравните (1) и (2). Ясно, что если (1) выполняется, то справедливым будет и (2), ведь если какое-то выражение больше или равно двум, то оно автоматически будет и больше нуля! Поэтому (2) можно и не решать.

Теперь посмотрим на простейшие нер-ва с корнем нечетной степени.

Пример. Найдите решение нер-ва

Решение. Всё очень просто – надо всего лишь возвести обе части в куб:

x 2 – 7x– 8 2 – 7x– 8 = 0

D = b 2 – 4ac = (– 7) 2 – 4•1•(– 8) = 49 + 32 = 81

Далее полученные точки отмечаются на координатной прямой. Они разобьют ее на несколько промежутков, на каждом из которых функция у =x 2 – 7x– 8 сохраняет свой знак. Определить же этот самый знак можно по направлению ветвей параболы, которую рисует схематично:

Видно, что парабола располагается ниже оси Ох на промежутке (– 1; 8). Поэтому именно этот промежуток и является ответом. Нер-во строгое, поэтому сами числа (– 1) и 8 НЕ входят в ответ, то есть для записи промежутка используются круглые скобки.

Обратите внимание: так как в исходном нер-ве используется корень нечетной (третьей) степени, то нам НЕ надо требовать, чтобы он был неотрицательным. Он может быть меньше нуля.

Теперь рассмотрим более сложный случай, когда в правой части нер-ва стоит не постоянное число, а некоторое выражение с переменной, то есть оно имеет вид

Случаи, когда n является нечетным числом, значительно более простые. В таких ситуациях достаточно возвести нер-во в нужную степень.

Пример. Решите нер-во

Решение.Слева стоит кубический корень, а возведем нер-во в третью степень (при этом мы используем формулу сокращенного умножения):

И снова квадратное нер-во. Найдем нули функции записанной слева, и отметим их на координатной прямой:

D = b 2 – 4ac = (– 1) 2 – 4•1•(– 2) = 1 + 8 = 9

Нер-во выполняется при х∈(– ∞; – 1)⋃(2; + ∞). Так как мы возводили нер-во в нечетную степень, то больше никаких действий выполнять не надо.

стоит корень четной степени, то ситуация резко осложняется. Его недостаточно просто возвести его в n-ую степень. Необходимо выполнение ещё двух условий:

f(x) > 0 (подкоренное выражение не может быть отрицательным);

g(x) > 0 (ведь сам корень должен быть неотрицательным, поэтому если g(x)будет меньше нуля, то решений не будет).

Вообще говоря, в таких случаях аналитическое решение найти возможно, но это тяжело. Поэтому есть смысл решить нер-во графически – такое решение будет более простым и наглядным.

Пример. Решите нер-во

Решение. Сначала решим его аналитически, без построения графиков. Возведя нер-во в квадрат, мы получим

х 2 – 10х + 21 > 0(1)

Решением этого квадратного нер-ва будет промежуток (– ∞;3)⋃(7; + ∞). Но надо учесть ещё два условия. Во-первых, подкоренное выражение должно быть не меньше нуля:

Во-вторых, выражение 4 – х не может быть отрицательным:

Получили ограничение 2,5 ⩽ х ⩽ 4, то есть х∈[2,5; 4]. С учетом того, что при решении нер-ва(1) мы получили х∈(– ∞;3)⋃(7; + ∞), общее решение иррационального нер-ва будет их пересечением, то есть промежутком [2,5; 3):

Скажем честно, что описанное здесь решение достаточно сложное для понимания большинства школьников, поэтому предложим альтернативное решение, основанное на использовании графиков. Построим отдельно графики левой и правой части нер-ва:

Видно, что график корня находится ниже прямой на промежутке [2,5; 3). Возникает вопрос – точно ли мы построили график? На самом деле с его помощью мы лишь определили, что искомый промежуток находится между двумя точками. В первой график корня касается оси Ох, а во второй точке он пересекается с прямой у = 4 – х. Найти координаты этих точек можно точно, если решить ур-ния. Начнем с первой точки:

Итак, координата х первой точки в точности равна 2,5. Для нахождения второй точки составим другое ур-ние:

Это квадратное ур-ние имеет корни 3 и 7 (убедитесь в этом самостоятельно). Число 7 является посторонним корнем:

Подходит только число 3, значит, вторая точка имеет координату х = 3, а искомый промежуток – это [2,5; 3).

Ещё тяжелее случаи, когда в нер-ве с корнем четной степени стоит знак «>», а не « 1/2 = х – 3

Дипломная работа: Методика решения иррациональных уравнений и неравенств в школьном курсе математики

§ 1. Анализ школьных учебников по алгебре и началам анализа

1.1. «Алгебра, 8», авт. А. Г. Мордкович

1.2. «Алгебра и начала анализа, 10-11», авт. А. Н. Колмогоров, А. М. Абрамов, Ю. П. Дудницин и др..

1.3. «Алгебра и начала анализа, 10-11», авт. Ш. А. Алимов, Ю. М. Колягин, Ю. В. Сидоров и др..

1.4. «Алгебра и начала анализа, 10-11», авт. М. И. Башмаков.

1.5. «Алгебра и начала анализа, 10-11», авт. А. Г. Мордкович.

1.6. «Сборник задач по алгебре, 8-9», авт. М. Л. Галицкий, А. М. Гольдман, Л. И. Звавич.

1.7. «Алгебра и математический анализ, 11», авт. Н. Я. Виленкин, О.С. Ивашев-Мусатов, С. И. Шварцбурд.

§ 2. Методика изучения иррациональных уравнений

2.1. Теоретические основы решения уравнений

2.1.1. Основные понятия, относящиеся к уравнениям

2.1.2. Наиболее важные приемы преобразования уравнений

2.2. Методы решения иррациональных уравнений

2.2.1. Метод сведения к эквивалентной системе уравнений и неравенств

2.2.2. Метод уединения радикала

2.2.3. Метод введения новой переменной.

2.2.4. Метод сведения к эквивалентным системам рациональных уравнений

2.2.5. Умножение обеих частей уравнения на функцию.

2.2.6. Решение иррациональных уравнений с использованием свойств входящих в них функций

3. Тождественные преобразования при решении иррациональных уравнений

§ 3. Методика решения иррациональных неравенств

3.1. Теоретические основы решения иррациональных неравенств

3.2. Методы решения иррациональных неравенств

3.2.1. Метод сведения к эквивалентной системе или совокупности рациональных неравенств

3.2.2. Умножение обеих частей неравенства на функцию

3.2.3. Метод введения новой переменной

3.2.4. Решение иррациональных неравенств с использованием свойств входящих в них функций

§ 4. Опытное преподавание

Материал, связанный с уравнениями и неравенствами, составляет значительную часть школьного курса математики. Одним из сложных разделов алгебры, изучаемых в школьной программе, являются иррациональные уравнения и неравенства, так как в школе им уделяют достаточно мало внимания.

Трудности при изучении данного вида уравнений и неравенств связаны со следующими их особенностями:

· в большинстве случаев отсутствие четкого алгоритма решения иррациональных уравнений и неравенств;

· при решении уравнений и неравенств данного вида приходится делать преобразования, приводящие к уравнениям (и неравенствам), не равносильным данному, вследствие чего чаще всего возникают ошибки, которые обычно связаны с потерей или приобретением посторонних корней в процессе решения.

Опыт показывает, что учащиеся в недостаточной степени овладевают умением решать иррациональные уравнения и неравенства, часто допускают ошибки при их решении. Однако задачи по теме «Иррациональные уравнения и неравенства» встречаются на вступительных экзаменах, и они довольно часто становятся «камнем преткновения».

Выше изложенное обусловило проблему исследования : обучение школьников решению иррациональных уравнений и неравенств, используя при этом основные методы решения иррациональных уравнений различных видов.

Объектом исследования является процесс обучения алгебре в 7-9 классах и алгебре и началам анализа в 10-11 классах.

Предметом исследования являются различные виды иррациональных уравнений и неравенств и методы их решения.

Целью работы является разработка методики изучения учащимися иррациональных уравнений и неравенств в школе.

Гипотеза исследования : освоение умения различать основные виды иррациональных уравнений и неравенств, умения применять необходимые приемы и методы их решения позволит учащимся решать иррациональные уравнения и неравенства на сознательной основе, выбирать наиболее рациональный способ решения, применять разные способы решения, в том числе те, которые не рассмотрены в школьных учебниках.

Для достижения поставленной цели и проверки гипотезы необходимо решить следующие задачи :

1. проанализировать действующие учебники алгебры и начала математического анализа для выявления представленной в них методики решения иррациональных уравнений и неравенств;

2. изучить стандарты образования по данной теме;

3. изучить статьи и учебно-методическую литературу по данной теме;

4. подобрать теоретический материал, связанный с равносильностью уравнений и неравенств, равносильностью преобразований, методами решения иррациональных уравнений и неравенств;

5. рассмотреть основные методы и приемы решения различных иррациональных уравнений и неравенств;

6. подобрать примеры решения иррациональных уравнений и неравенств для демонстрации излагаемой теории;

8. осуществить опытное преподавание.

При изучении любой новой темы в основном курсе школы встает проблема изложения данной темы в школьных учебниках. Пропедевтикой изучения раздела иррациональных уравнений и неравенств в школе является введение понятие арифметического корня и, соответственно, рассмотрение его свойств.

Проанализируем в каких классах вводится данное понятие разными авторами учебников. Алимов Ш. А. в учебнике «Алгебра. 9класс» вводит понятие арифметического корня натуральной степени, а также свойства арифметического корня. Макарычев Н. Г. же разделяет понятия квадратного корня и корня -ой степени. В учебнике «Алгебра. 8 класс» классе вводится понятие арифметического квадратного корня и, соответственно, рассматриваются его свойства. В учебнике «Алгебра. 9 класс» вводятся понятия корня -ой степени, арифметического корня -ой степени и рассматриваются свойства арифметического корня -ой степени. Колмогоров А. Н. в учебнике «Алгебра. 10 класс» вводит понятия корня -ой степени, арифметического корня -ой степени и рассматривает свойства арифметического корня -ой степени перед изучением иррациональных уравнений. Мордкович А. Г. в учебнике «Алгебра. 8 класс» вводит понятие квадратного корня и его свойства. Кроме того, в этом же учебнике есть отдельный параграф, посвященный иррациональным уравнениям.

1.1. «Алгебра, 8», авт. А. Г. Мордкович [27], [28]

Данное учебное пособие состоит из двух частей: учебника и задачника.

В I части данного учебного пособия материал, посвященный иррациональным уравнениям, изложен в главе «Квадратные уравнения» в параграфе «Иррациональные уравнения». Параграф начинается с определения иррационального уравнения. Далее рассматривается решение иррационального уравнения по определению квадратного корня из чего выводится метод решения иррациональных уравнений – метод возведения в квадрат обеих частей уравнения. Затем данный метод демонстрируется на примерах решения иррациональных уравнений вида , . Найденные корни проверяются подстановкой в исходное уравнение, при этом обращено внимание на те случаи, когда могут появиться посторонние корни. Автор подчеркивает, что проверка – обязательный этап решения иррационального уравнения. Далее приводится решение уравнения вида методом введения новой переменной . Параграф завершается беседой о равносильных и неравносильных преобразованиях: дается определение равносильных уравнений, перечисляются и демонстрируются на примерах равносильные и неравносильные преобразования.

Система задач во II части данного учебного пособия достаточно разнообразна. В №№ 1011-1014 необходимо решить иррациональные уравнения вида , где – линейное, квадратное или дробно-рациональное выражение. В № 1015 чтобы решить уравнение необходимо сначала уединить радикал. В № 1016 для решения предложены уравнения вида . №№ 10017-1020 –упражнения для решения методом замены иррациональных уравнений вида , , . В №№ 1023, 1024 необходимо выяснить, равносильны ли уравнения. В №№ 1021, 1022, 1025-1027 нужно решить уравнения вида , , где выражения , могут быть как линейными так и квадратными, а в №№ 1028-1031 – уравнения вида .

№№ 1032, 1033 – упражнения повышенной трудности для решения иррациональных уравнений методом замены.

1.2. «Алгебра и начала анализа, 10-11», авт. А. Н. Колмогоров, А. М. Абрамов, Ю. П. Дудницин и др. [13].

Материал по данной теме изложен в IV главе «Показательная и логарифмическая функции», как пункт «Иррациональные уравнения» параграфа «Обобщение понятия степени». Автор рекомендует рассматривать решение иррациональных уравнений в теме «Уравнения, неравенства, системы», где систематизируются сведения об уравнениях.

В пункте «Иррациональные уравнения» дается понятие иррационального уравнения, приводится несколько примеров простейших иррациональных уравнений вида , которые решаются с помощью возведения обеих частей уравнения в квадрат. Найденные корни проверяются подстановкой в исходное уравнение, при этом обращено внимание на те случаи, когда могут появиться посторонние корни. Показано, что кроме возведения в квадрат иррациональные уравнения удобно решать, используя равносильный переход от уравнения к системе, состоящей из уравнения и неравенства. Рассмотрен пример иррационального уравнения, содержащего корень третьей степени. Для того чтобы «избавиться от радикала», обе части такого уравнения возводятся в куб.

После пункта приведены упражнения для закрепления умений решать иррациональные уравнения. В №№417-420 предложены простейшие уравнения вида , решить которые можно с помощью возведения обеих частей уравнения либо в квадрат, либо в куб, а также используя равносильные переходы. Такие задачи, по мнению авторов учебника необходимо уметь решать для получения удовлетворительной оценки. Задачи же в №№422-425 чуть сложнее. Здесь уравнения содержат корни выше третьей степени.

Иррациональным неравенствам в данном пункте внимания не уделено.

В заключительной главе учебника «Задачи на повторение» помещены практические упражнения для повторения курса. Здесь в параграфе «Уравнения, неравенства, системы уравнений и неравенств» иррациональным уравнениям и неравенствам посвящен пункт «Иррациональные уравнения и неравенства». То есть, не смотря на то, что в основной части учебника иррациональным неравенствам внимания не уделено, автор включает в задания для повторения такие неравенства.

1.3. «Алгебра и начала анализа, 10-11», авт. Ш. А. Алимов, Ю. М. Колягин, Ю. В. Сидоров и др. [1].

В данном учебнике нет материала, посвященного иррациональным уравнениям и неравенствам. Лишь в конце ученика помещены упражнения для итогового повторения курса алгебры. Здесь есть только один номер для решения простейших иррациональных уравнений (№801). Упражнений для решения иррациональных неравенств нет.

Это можно объяснить тем, что, по мнению автора, умение решать иррациональные неравенства не является обязательным для учащихся и соответствующая тема может быть предложена для изучения самостоятельно или на факультативных занятиях. [14] Поэтому в учебнике предложены задачи для внеклассной работы, где встречаются иррациональные уравнения (№№934, 947) и неравенства (№942).

1.4. «Алгебра и начала анализа, 10-11», авт. М. И. Башмаков [2].

В данном учебном пособии иррациональные уравнения и неравенства рассматриваются в заключительной VI главе «Уравнения и неравенства». Глава предназначена для систематизации и обобщения сведений об уравнениях, неравенствах и системах уравнений. В начале главы помещена вводная беседа, которая состоит из трех пунктов.

В пункте «Уравнение» вводятся такие понятия как уравнение, неизвестные, корень уравнения, подробно рассказывается, что значит решить уравнение с одним или двумя неизвестными, что означает найти корни уравнения, приведены некоторые рекомендации о форме записи ответа при решении уравнений с одним или двумя неизвестными.

В пункте «Равносильность» выясняется, когда одно уравнение является следствием другого, вводится понятие равносильных уравнений. Автор подробно останавливается на некоторых полезных преобразованиях уравнений:

1) Перенос членов из одной части уравнения в другую с противоположным знаком.

2) Переход к совокупности уравнений.

3) Переход к системе уравнений.

Все равносильные переходы представлены в виде схем и рассмотрены на примерах.

В следующем пункте «Неравенство» приведены примеры верных и неверных числовых неравенств, основные правила преобразования неравенств, при этом используются знаки следствия и равносильности. Вводятся такие понятия как ОДЗ неравенства, решение неравенства, равносильные неравенства, выясняется, когда одно неравенство является следствием другого.

§1 «Уравнения с одним неизвестным» состоит из трех пунктов: «Общие приемы», «Примеры решения уравнений» и «Приближенные методы вычисления корней». В первом пункте перечислены стандартные уравнения, которые были изучены ранее. Основным шагом в решении уравнения является преобразование уравнения к одному из стандартных. Приведены некоторые наиболее употребительные приемы, общие для всех типов уравнений:

1) Разложение на множители.

2) Введение нового неизвестного.

3) Графический метод.

Отметим, что во втором пункте на ряду со стандартными уравнениями рассматривается решения только одного простейшего иррационального уравнения с помощью равносильного перехода к системе.

В третьем пункте кратко рассказывается о таких методах приближенного вычисления корней как метод половинного деления, метод хорд и касательных.

§ 2 «Неравенства с одним неизвестным» состоит из двух пунктов: «Общие приемы» и «Примеры решения неравенств». В первом пункте демонстрируется два приема решения неравенств: разложение на множители и метод замены неизвестного.

Во втором пункте на примерах показана техника решения неравенств с помощью переходов, сохраняющих равносильность. Отметим, что на ряду со стандартными неравенствами рассматривается решение только одного простейшего иррационального неравенства.

В конце главы помещены задания для решения иррациональных уравнений №17, для решения иррациональных неравенств – №21, в котором есть задание со звездочкой, то есть относящееся к разделу «трудные задачи».

Иррациональным уравнениям и неравенствам в главе уделено недостаточно внимания: приведены решения с помощью переходов, сохраняющих равносильность одного простейшего иррационального уравнения и одного неравенства.

Цель данной главы – обобщить имеющиеся у учащихся знаний об уравнениях, неравенствах и системах уравнений, поэтому здесь подробно не рассматриваются конкретные виды уравнений, а лишь повторяются сведения об изученных видах уравнений и методах их решения. [14]

Данное учебное пособие состоит из двух частей: учебника и задачника.

В I части данного учебного пособия материал, касающийся иррациональных уравнений и неравенств, изучается в последней VIII главе «Уравнения и неравенства. Системы уравнений и неравенств», завершающей изучение школьного курса алгебры и начал математического анализа. Здесь уравнения и неравенства рассматриваются с самых общих позиций. Это, с одной стороны, своеобразное подведение итогов и, с другой стороны, некоторое расширение и углубление знаний.

В первых трех параграфах этой главы подведены итоги изучения в школе уравнений, неравенств. Использованы следующие термины :

¨ равносильность уравнений, равносильность неравенств;

¨ следствие уравнения, следствие неравенства;

¨ равносильное преобразование уравнения, неравенства;

¨ посторонние корни (для уравнений);

¨ проверка корней (для уравнений).

¨ о равносильности уравнений;

¨ о равносильности неравенств.

Даны ответы на четыре главных вопроса , связанных с решением уравнений:

1) как узнать, является ли переход от одного уравнения к другому равносильным преобразованием;

2) какие преобразования переводят данное уравнение в уравнение-следствие;

3) как сделать проверку, если она сопряжена со значительными трудностями в вычислениях;

4) в каких случаях при переходе от одного уравнения к другому может произойти потеря корней и как этого не допустить?

Перечислены возможные причины расширения области определения уравнения, одна из которых – освобождение в процессе решения уравнения от знаков корней четной степени; указаны причины, по которым может произойти потеря корней при решении уравнений.

Выделены четыре общих метода решения уравнений:

2) метод разложения на множители;

3) метод введения новых переменных;

4) функционально-графический метод.

Что касается иррациональных уравнений, то им в данном учебном пособии уделено достаточно большое внимание.

На примере иррационального уравнения показано как решение любого уравнения осуществляется в три этапа: технический , анализ решения, проверка.

Также на примере иррационального уравнения показано, как сделать проверку, если проверка корней с помощью их подстановки в исходное уравнение сопряжена со значительными вычислительными трудностями.

Метод замены уравнения h (f (x ))=h (g (x )) уравнением f (x )=g (x ) применятся при решении иррациональных уравнений для перехода от уравнения к уравнению .

Метод введения новой переменной также разобран и на примере решения иррационального уравнения.

Отдельный пункт посвящен иррациональным неравенствам. Здесь с теоретическим обоснованием рассматривается решение неравенств вида , . В первом случае иррациональное неравенство заменяется равносильной системой неравенств во втором – равносильной совокупностью систем неравенств

Система задач во II части данного учебного пособия изложена в той же последовательности, что и соответствующий материал в I части. В § 55 «Равносильность уравнений» изложены различные типы заданий на равносильность и следствие уравнений, в том числе и иррациональных. В § 56 «Общие методы решения уравнений» помещены задания для использования четырех методов, изложенных в I части данного учебного пособия, для решения уравнений. Все задачи в соответствии с ними разбиты на четыре блока, в каждом из которых встречаются иррациональные уравнения. В § 57 «Решение неравенств с одной переменной» изложены различные типы заданий на равносильность и следствие неравенств, в том числе и иррациональных.

В № 1673 нужно решить простейшие иррациональные уравнения. №№1674, 1675, 1712-1719 – упражнения выше среднего уровня для решения иррациональных уравнений, №№1790, 1791 – неравенств. № 1792 – упражнение повышенной трудности для решения иррациональных неравенств.

Много заданий, в которых требуется решить «смешанное» уравнение или неравенство, то есть логарифмическое, показательное или тригонометрическое уравнение или неравенство, в которое входят и иррациональные выражения. Среди этих заданий есть задания как базового, так и повышенного уровня.

В I части учебника много внимание уделено равносильности уравнений и неравенств, достаточно строго рассмотрены общие методы решения уравнений, с оговоркой о потере корней и приобретении посторонних. II часть учебника отличается обилием и разнообразием задач. Достаточно много задач на равносильность и следствие уравнений и неравенств.

1.6. «Сборник задач по алгебре, 8-9», авт. М. Л. Галицкий, А. М. Гольдман, Л. И. Звавич [5].

Данная книга представляет собой сборник задач по курсу алгебры, предназначенный для учащихся 8-9 классов с углубленным изучением математики.

В начале параграфа «Степень с рациональным показателем» помещен справочный материал теоретического характера, посвященный иррациональным уравнениям и неравенствам. Описаны такие пути решения иррациональных уравнений, как:

· возведение обеих частей уравнения в натуральную степень с последующей проверкой найденных корней;

· переход к равносильным системам, в которых учитывается область определения уравнения и требование того, что бы были неотрицательными обе части уравнения, возводимые в четную степень.

При решении иррациональных неравенств либо используется метод интервалов, либо с помощью равносильных преобразований заменяется данное иррациональное неравенство системой (или совокупностью систем) рациональных неравенств.

В параграфе рассмотрено три способа решения иррационального уравнения вида :

1) переход к равносильной системе;

2) введение новой переменной;

3) использование свойства монотонности функций.

Среди упражнений, помещенных в данном параграфе, есть упражнения для закрепления умений и навыков решать иррациональные уравнения и неравенства. В №№115-117 необходимо доказать, что уравнение не имеет решения, в №№118-119 – ответить на вопрос: равносильны ли уравнения. №№120-144 предлагаются для решения иррациональных уравнений, №№145-155 – для решения неравенств описанными выше способами.

1.7. «Алгебра и математический анализ, 11», авт. Н. Я. Виленкин, О.С. Ивашев-Мусатов, С. И. Шварцбурд [4].

Данное учебное пособие представляет собой продолжение книги «Алгебра и начала анализа» для 10 класса и предназначено как для общеобразовательной школы, так и классов и школ с углубленным изучением курса математики.

Иррациональные уравнения и неравенства изучаются в параграфе «Степенная функция. Иррациональные выражения, уравнения и неравенства» VIII главы «Показательная, логарифмическая и степенные функции».

Пункт «Иррациональные уравнения» начинается с определения иррационального уравнения и примеров таких уравнений. Далее сформулирована и доказана теорема о равносильных уравнениях, на которой основано решение иррациональных уравнений. Из теоремы следует, что если в ходе решения иррационального уравнения приходилось возводить обе его части в степень с четным показателем, то могут появиться посторонние корни. Поэтому, чтобы не было необходимости подставлять найденные корни в данное уравнение, сформулировано еще два утверждения о равносильном переходе от уравнений вида и к системам, состоящим из уравнения и неравенства. Далее на примерах решения иррациональных уравнений демонстрируются данные равносильные переходы. Также автор рекомендует перед возведением обеих частей уравнения в некоторую степень «уединить радикал», то есть представить уравнение в виде . Далее данный метод применяется для решения иррациональных уравнений

После данного пункта помещены упражнения для закрепления умений решать иррациональные уравнения описанными выше методами – №216. В №215 необходимо доказать, что данные иррациональные уравнения не имеют решений.

В следующем пункте «Иррациональные неравенства» сформулированы приемы решения иррациональных неравенств вида и с помощью равносильного перехода к системе неравенств в первом случае и совокупности систем неравенств – во втором. Рассматривается решение иррационального неравенства вида с помощью равносильного перехода к неравенству . Решение каждого из видов неравенств демонстрируется на примерах.

После данного пункта помещены упражнения (№217) для закрепления умения решать иррациональные неравенства с помощью равносильных переходов, описанных выше.

Все утверждения, сформулированные в данном учебном пособии, изложены со строгим обоснованием. Описан полезный метод при решении иррациональных уравнений – метод «уединения радикала». Не смотря на то, что учебник не отличается обилием упражнений, предлагаемые задания разнообразны, различной степени сложности

Проведенный анализ позволяет сделать следующие выводы:

1) В учебнике [1] материала по методам решения иррациональных уравнений нет. В учебниках [13] и [4] материала по теории способов решения иррациональных уравнений достаточно. В большом объеме теория по общим методам решения рассмотрена учебнике [2] и [10].

2) В каждом учебнике рассмотрены два основных способа решения: возведение обеих частей уравнения в степень, с последующей подстановкой полученных корней в исходное уравнение, а также решение уравнений с помощью равносильных переходов к системе, состоящей из уравнения и неравенства. В учебниках [2] и [10] рассмотрены такие общие методы решения уравнений как метод разложения на множители, метод введения новых переменных, функционально-графический метод; некоторые из них продемонстрированы на примерах решения иррационального уравнения.

3) В учебниках [1] и [13] не рассмотрено решение иррациональных неравенств. В учебнике [2] материала по решению иррациональных неравенств не достаточно. В учебниках [4] и [10] подробно и с теоретическим обоснованием рассмотрено решение иррациональных неравенств вида , с помощью равносильного перехода к системе (или совокупности систем). Только в учебнике [4] рассматривается решение иррационального неравенства вида .

4) Наиболее большой объем упражнений для решения иррациональных уравнений и неравенств представлен в учебниках [11] и [5]. В учебнике [4] упражнений немного, но они разнообразны.

2.1. Теоретические основы решения уравнений

2.1.1. Основные понятия, относящиеся к уравнениям

, (1)

где и – некоторые функции, называют уравнением с одним неизвестным x (с одной переменной x ). Это равенство может оказаться верным при одних значениях x и неверным при других значениях x .

Число a называется корнем (или решением ) уравнения (1), если обе части уравнения (1) определены при и равенство является верным. Следовательно, каждый корень уравнения (1) принадлежит множеству, которое является пересечением (общей частью) областей определения функций и и называется областью допустимых значений (ОДЗ) уравнения (1).

Решить уравнение – значит найти все его корни или доказать, что корней нет.

Если в условиях задачи не указано, на каком множестве нужно решить уравнение, то решение следует искать в ОДЗ этого уравнения.

В процессе решения часто приходится преобразовывать уравнение, заменяя его более простым (с точки зрения нахождения корней). Есть одно правило, которое не следует забывать при преобразовании уравнений: нельзя выполнять преобразования, которые могут привести к потере корней .

Назовем преобразование уравнения (1) допустимым , если при этом преобразовании не происходит потери корней, то есть получается уравнение

, (2)

которое либо имеет те же корни, что и уравнение (1), либо, кроме всех корней уравнения (1), имеет хотя бы один корень, не являющийся корнем уравнения (1), посторонний для уравнения (1) корень. В связи с этим используют следующие понятия.

Уравнение (2) называется следствием уравнения (1), если каждый корень уравнения (1) является корнем уравнения (2).

Уравнения (1) и (2) называются равносильными (эквивалентными), если каждое из этих уравнений является следствием другого. Иными словами, уравнения (1) и (2) равносильны, если каждый корень уравнения (1) является корнем уравнения (2) и наоборот, каждый корень уравнения (2) является корнем уравнения (1). Уравнения, не имеющие корней, считаются равносильными.

Если уравнения (1) и (2) равносильны, то пишут

или (1)(2),

а если уравнение (2) является следствием уравнения (1), то пишут

или (1)(2).

Отметим, что если исходное уравнение с помощью допустимых преобразований заменено другим, причем в процессе преобразования хотя бы один раз уравнение заменялось неравносильным ему следствием, то проверка найденных корней путем подстановки в исходное уравнение является обязательной .

Если же при каждом преобразовании уравнение заменялось равносильным, то проверка не нужна (не следует путать проверку с контролем вычислений).

Рассмотрим еще одно понятие, связанное с решением уравнений. Будем говорить, что уравнение (1) равносильно совокупности уравнений

, (3)

если выполнены следующие условия:

1) каждый корень уравнения (1) является корнем, по крайней мере, одного из уравнений (3);

2) любой корень каждого из уравнений (3) является корнем уравнения (1).

Если указанные условия выполнены, то множество корней уравнения (1) является объединением множеств корней уравнений (3).

Если уравнение записано в виде

, (4)

то каждое решение этого уравнения является решением, по крайней мере, одного из уравнений

(5)

Однако нельзя утверждать, что любой корень каждого из уравнений (5) есть корень уравнения (4).

Например, если , то – корень уравнения , но число 3 не является корнем уравнения (4), так как функция не определена при .

Таким образом, в общем случае нельзя утверждать, что уравнение (4) равносильно совокупности уравнений (5). Чтобы решить уравнение (4), достаточно найти корни уравнений и , а затем отбросить те, которые не входят в ОДЗ уравнения (4), то есть не принадлежат множеству, на котором определены функции и . В ОДЗ уравнения (4) это уравнение равносильно совокупности уравнений (5). Справедливо более общее утверждение: если функция определена при всех x таких, что , а функция определена при всех x таких, что , то уравнение (4) равносильно совокупности уравнений (5). [18]

2.1.2. Наиболее важные приемы преобразования уравнений

Все преобразования уравнений можно разделить на два типа: [15]

1) Равносильные, то есть преобразования, после применения любых из которых получится уравнение, равносильное исходному.

2) Неравносильные, то есть преобразования, после применения которых может произойти потеря или приобретение посторонних корней.

Рассмотрим некоторые виды преобразований уравнений и проанализируем, к каким типам они относятся.

1. Перенос членов уравнения из одной части в другую , то есть переход от уравнения

(1)

. (2)

Указанное преобразование приводит к равносильному уравнению, то есть (1)(2).

В частности, . Заметим, что здесь речь идет только о переносе членов уравнения из одной его части в другую без последующего приведения подобных членов (если таковые имеются). [18]

2. Приведение подобных членов , то есть переход от уравнения

(3)

. (4)

Справедливо следующее утверждение: для любых функций ,, уравнение (4) является следствием уравнения (3), то есть (3)(4).

Переход от уравнения (3) к уравнению (4) является допустимым преобразованием, при котором потеря корней невозможна, но могут появиться посторонние корни.

Таким образом, при приведении подобных членов, а также при отбрасывании одинаковых слагаемых в левой и правой частях уравнения получается уравнение, являющееся следствием исходного уравнения. [18]

Например, если в уравнении

вычеркнуть в левой и правой его частях слагаемое , то получится уравнение

,

являющееся следствием исходного: второе уравнение имеет корни , , а первое – единственный корень .

Отметим еще, что если ОДЗ уравнения (4) содержится в области определения функции , то уравнения (3) и (4) равносильны.

3. Умножение обеих частей уравнения на одну и ту же функцию , то есть переход от уравнения (4) к уравнению

. (5)

Справедливы следующие утверждения:

1) если ОДЗ уравнения (4), то есть пересечение областей определения функций и , содержится в области определения функции , то уравнение (5) является следствием уравнения (4);

2) если функция определена и отлична от нуля в ОДЗ уравнения (4), то уравнения (4) и (5) равносильны. [18]

Заметим, что в общем случае переход от уравнения (5) к уравнению (4) недопустим, так как это может привести к потере корней.

При решении уравнений вида (5) обычно заменяют его равносильным уравнением

,

затем находят все корни уравнений

и

и, наконец, проверяют, какие из этих корней удовлетворяют уравнению (5).

4. Возведение обеих частей уравнения в натуральную степень , то есть переход от уравнения

(6)

. (7)

Справедливы следующие утверждения:

1) при любом уравнение (7) является следствием уравнения (6);

2) если (n – нечетное число), то уравнения (6) и (7) равносильны;

3) если (n – четное число), то уравнение (7) равносильно уравнению

, (8)

а уравнение (8) равносильно совокупности уравнений

. (9)

В частности, уравнение

(10)

равносильно совокупности уравнений (9). [18]

Следовательно, исходя из утверждений 1 и 2, возведение обеих частей уравнения в нечетную степень и извлечение из обеих частей уравнения корня нечетной степени является равносильным преобразованием.

Исходя из утверждения 1 и 3, возведение обеих частей уравнения в четную степень и извлечение из обеих частей уравнения корня четной степени является неравносильным преобразованием, при этом получается уравнение, являющееся следствием исходного.

5. Применение формулы при является равносильным преобразованием, при – неравносильным. [15], [18]

Преобразования уравнений, рассмотренные в пунктах 3, 4 и 5 будут продемонстрированы на примерах ниже.

2.2. Методы решения иррациональных уравнений

В работе будем придерживаться следующего определения иррационального уравнения:

Иррациональным уравнением называется уравнение, содержащее неизвестное под знаком корня.

Прежде чем приступить к решению сложных уравнений учащиеся должны научиться решать простейшие иррациональные уравнения. К простейшим иррациональным уравнениям относятся уравнения вида: .

Основная идея решения иррационального уравнения состоит в сведении его к рациональному алгебраическому уравнению, которое либо равносильно исходному иррациональному уравнению, либо является его следствием.

Главный способ избавиться от корня и получить рациональное уравнение – возведение обеих частей уравнения в одну и ту же степень, которую имеет корень, содержащий неизвестное, и последующее «освобождение» от радикалов по формуле . [6]

Если обе части иррационального уравнения возвести в одну и ту же нечетную степень и освободиться от радикалов, то получится уравнение, равносильное исходному. [6]

При возведении уравнения в четную степень получается уравнение, являющееся следствием исходного. Поэтому возможно появление посторонних решений уравнения, но не возможна потеря корней. Причина приобретения корней состоит в том, что при возведении в четную степень чисел, равных по абсолютной величине, но разных по знаку, получается один и тот же результат.

Так как могут появиться посторонние корни, то необходимо делать проверку , подставляя найденные значения неизвестной только в первоначальное уравнение, а не в какие-то промежуточные.

Рассмотрим применение данного метода для решения иррациональных уравнений вида . [7]

Пример 1 . Решить уравнение .

Решение. Возведем обе части этого уравнения в квадрат и получим , откуда следует, что или .

Проверка. : . Это неверное числовое равенство, значит, число не является корнем данного уравнения.

: . Это верное числовое равенство, значит, число является корнем данного уравнения.

Ответ. .

Пример 2 . Решить уравнение .

Решение. После возведения в квадрат получаем уравнение , откуда следует что или .

Проверка . : . Это верное числовое равенство, значит, число является корнем данного уравнения.

: . Это неверное числовое равенство, значит, число не является корнем данного уравнения.

Ответ. .

2.2.1. Метод сведения к эквивалентной системе уравнений и неравенств

Проверка, осуществляемая подстановкой найденного решения в исходное уравнение, может быть легко реализована, если проверяемые корни – «хорошие» числа, а для «громоздких» корней проверка может быть сопряжена со значительными вычислительными трудностями. Поэтому каждый образованный школьник должен уметь решать иррациональные уравнения с помощью равносильных преобразований, так как, выполняя равносильные преобразования, можно не опасаться ни потери корней, ни приобретения посторонних решений. [17]

Аккуратное возведение в четную степень уравнения вида состоит в переходе к равносильной ему системе :

Неравенство в этой системе выражает условие, при котором уравнение можно возводить в четную степень, отсекает посторонние решения и позволяет обходиться без проверки. [17]

Школьники довольно часто добавляют к этой системе неравенство . Однако этого делать не нужно и даже опасно, поскольку условие автоматически выполняется для корней уравнения , в правой части которого стоит неотрицательное выражение. [9]

Пример 3 . Решить уравнение .

Решение. Это уравнение равносильно системе

Решая первое уравнение этой системы, равносильное уравнению , получим корни и .

Второй корень не удовлетворяет неравенству системы и, следовательно, является посторонним корнем исходного уравнения.

Ответ. .

Полезно запомнить схему решения еще одного вида иррациональных уравнений . Такое уравнение равносильно каждой из двух систем

Поскольку после возведения в четную степень получаем уравнение-следствие . Мы должны, решив его, выяснить, принадлежат ли найденные корни ОДЗ исходного уравнения, то есть выполняется ли неравенство (или ). На практике из этих систем выбирают для решения ту, в которой неравенство проще. [9]

Пример 4 . Решить уравнение .

Решение. Это уравнение равносильно системе

Решая первое уравнение этой системы, равносильное уравнению , получим корни и . Однако при этих значениях x не выполняется неравенство , и потому данное уравнение не имеет корней.

Ответ . Корней нет.

2.2.2. Метод уединения радикала

При решении иррациональных уравнений полезно перед возведением обеих частей уравнения в некоторую степень «уединить радикал» , то есть представить уравнение в виде . Тогда после возведения обеих частей уравнения в n ую степень радикал справа исчезнет. [4]

Пример 5 . Решить уравнение

Решение . Метод уединения радикала приводит к уравнению . Это уравнение равносильно системе

Решая первое уравнение этой системы, получим корни и , но условие выполняется только для .

Ответ. .

Пример 6 . Решить уравнение .

Решение . Уединив первый радикал, получаем уравнение

,

Возводя обе части этого уравнения в квадрат, получаем уравнение

,

.

Последнее уравнение является следствием исходного уравнения. Возводя обе части этого уравнения в квадрат, приходим к уравнению

,.

Это уравнение является следствием уравнения исходного уравнения и имеет корни , . Первый корень удовлетворяет исходному уравнению, а второй – не удовлетворяет.

Ответ . .

2.2.3. Метод введения новой переменной.

Мощным средством решения иррациональных уравнений является метод введения новой переменной, или «метод замены». Метод обычно применяется в случае, если в уравнении неоднократно встречается некоторое выражение , зависящее от неизвестной величины. Тогда имеет смысл обозначить это выражение какой-нибудь новой буквой и попытаться решить уравнение сначала относительно введенной неизвестной, а потом уже найти исходную неизвестную. В ряде случаев удачно введенные новые неизвестные иногда позволяют получить решение быстрее и проще; иногда же без замены решить задачу вообще невозможно. [6], [17]

Пример 7 . Решить уравнение .

Решение . Положив , получим существенно более простое иррациональное уравнение . Возведем обе части уравнения в квадрат: .

Далее последовательно получаем:

;

;

;

;

, .

Проверка найденных значений их подстановкой в уравнение показывает, что – корень уравнения, а – посторонний корень.

Возвращаясь к исходной переменной x , получаем уравнение , то есть квадратное уравнение , решив которое находим два корня: ,. Оба корня, как показывает проверка, удовлетворяют исходному уравнению.

Ответ : , .

Замена особенно полезна, если в результате достигается новое качество, например, иррациональное уравнение превращается в квадратное.

Пример 8 . Решить уравнение .

Решение. Перепишем уравнение так: .

Видно, что если ввести новую переменную , то уравнение примет вид , откуда , .

Теперь задача сводится к решению уравнения и уравнения . Первое из этих решений не имеет, а из второго получаем , . Оба корня, как показывает проверка, удовлетворяют исходному уравнению.

Ответ . , .

Отметим, что «бездумное» применение в Примере 8 метода «уединения радикала» и возведение в квадрат привело бы к уравнению четвертой степени, решение которого представляет собой в общем случае чрезвычайно сложную задачу.

Пример 9 . Решить уравнение .

Введем новую переменную

, .

В результате исходное иррациональное уравнение принимает вид квадратного

,

откуда учитывая ограничение , получаем . Решая уравнение , получаем корень . Как показывает проверка, удовлетворяет исходному уравнению.

Ответ . .

Иногда посредством некоторой подстановки удается привести иррациональное уравнение к рациональному виду, как рассмотренных Примерах 8, 9. В таком случае говорят, что эта подстановка рационализирует рассматриваемое иррациональное уравнение, и называют ее рационализирующей., основанный на применении рационализирующих подстановок, называется способом рационализации .

Со всеми учащимися на уроке этот способ решения иррациональных уравнений разбирать не нужно, но он может быть рассмотрен в рамках факультативных или кружковых занятий по математике с учащимися, проявляющих повышенный интерес к математике.

2.2.4. Метод сведения к эквивалентным системам рациональных уравнений

Уравнения вида (здесь a , b , c , d некоторые числа, m , n натуральные числа) и ряд других уравнений часто удается решить при помощи введения двух вспомогательных неизвестных: и , где и последующего перехода к эквивалентной системе рациональных уравнений . [17]

Пример 1 6 . Решить уравнение .

Решение . Введем новые переменные

и , где .

Тогда исходное уравнение принимает вид: . Полученное уравнение обладает одним существенным недостатком: в нем две неизвестных. Но заметим, что величины y и z не являются независимыми переменными – они зависят одна от другой посредством старой переменной x . Выразим x через y и z : и . Теперь, можно заметить, что если первое уравнение умножить на два и затем вычесть из него второе, то переменная x исключается, и остается связь только между y и z

.

В результате получаем систему двух уравнений относительно двух неизвестных y и z

Решая эту систему методом подстановки, приходим к уравнению , корнями которого являются числа и . Корень посторонний, поскольку . Осталось решить уравнение , откуда находим .

Ответ . .

Пример 1 7 . Решить уравнение . [6]

Решение . Возведение обеих частей этого уравнения в четвертую степень не обещает ничего хорошего. Если же положить , , то исходное уравнение переписывается так: . Поскольку мы ввели две новые неизвестные, надо найти еще одно уравнение, связывающее y и z . Для этого возведем равенства , в четвертую степень и заметим, что .

Итак, надо решить систему уравнений

она имеет два (действительных) решения: , ; , .

Остается решить систему двух уравнений с одним неизвестным

первая из них дает , вторая дает .

Ответ : , .

Не всегда после введения новых переменных удается исключить неизвестную x , как это было в рассмотренных Примерах 15, 16 . Однако, как можно убедиться из следующего примера, переход от уравнения к системе может помочь и в таком случае. [17]

Пример 1 8 . Решить уравнение .

Решение. Введем новые переменные

и , где .

По стандартной схеме получим следующую систему уравнений:

откуда следует, что

.

Так как , то y и z должны удовлетворять системе

Возведем оба уравнения этой системы в квадрат, после чего, сложив их, получаем уравнение .

Также возведем равенства , в квадрат и заметим, что .

Получаем следующую систему уравнений:

из которой получаем уравнение .

Заметим, что это уравнение имеет корень . Тогда, разделив многочлен на , получаем разложение левой части уравнения на множители

.

Отсюда следует, что – единственное решение этого уравнения. После проверки записываем это решение в ответ.

Ответ : .

Иногда иррациональное уравнение удается решить довольно быстро, если обе его части умножить на удачно подобранную функцию. Конечно, при умножении обеих частей уравнения на некоторую функцию могут появиться посторонние решения, ими могут оказаться нули самой этой функции. Поэтому предлагаемый метод требует обязательного исследования получающихся значений. [6]

Пример 19. Решить уравнение .

Решение . Умножим обе части уравнения на одну и ту же функцию . Выражение называется сопряженным для выражения . Цель такого умножения ясна: использовать тот факт, что произведение двух сопряженных выражений уже не содержит радикалов.

В результате этого умножения и очевидных преобразований приходим к уравнению

,

которое равносильно совокупности уравнений

Уединив первый радикал второго уравнения совокупности, возведем его в квадрат и получим

Если внимательно посмотреть на неравенства последней системы, можно заметить, что пересечение множеств и пусто. Следовательно, уравнение решений не имеет. Значит, уравнение имеет единственный корень .

Подстановка в исходное уравнение показывает, что – корень.

Ответ : .

Впрочем, здесь можно было обойтись и без подстановки: функция нигде в нуль не обращается, и поэтому умножение обеих частей уравнения на эту функцию не приводит к появлению посторонних решений.

Пример 20. Решить уравнение . [9]

Решение . Умножим обе части уравнения на функцию . После преобразований получим уравнение

.

Оно имеет два корня: . Проверка показывает, что – посторонний корень (нетрудно видеть, – корень функции ). Таким образом, уравнение имеет единственный корень .

Ответ : .

2.2.6. Решение иррациональных уравнений с использованием свойств входящих в них функций

В школьном курсе математики изучаются свойства многих элементарных функций. Их иногда с успехом можно применять и при решении иррациональных уравнений. Рассмотрим несколько примеров.

1. Использование монотонности функции.

Если уравнение имеет вид

где возрастает (убывает), или

где и «встречно монотонны», т.е. возрастает, а убывает и наоборот, то такое уравнение имеет не более одного корня. Если удается заметить это или привести уравнение к такому виду и при этом нетрудно угадать корень, то он и будет решением данного уравнения. [9]

Пример 21 . .

Решение . Это уравнение можно попытаться решить возведением в квадрат (трижды!). Однако при этом получится уравнение четвертой степени. Попробуем угадать корень. Это сделать нетрудно: . Теперь заметим, что левая часть уравнения – возрастающая функция, а правая – убывающая. Но это значит, что больше одного корня такое уравнение иметь не может. Итак, – единственный корень.

Ответ : .

Пример 22 . Решить уравнение .

Решение . Традиционный метод решения уравнений такого вида хорошо известен. Впрочем, легко заметить, что – корень. Левая часть уравнения задает возрастающую функцию, правая – константу. Следовательно, данное уравнение может иметь не более одного корня. Итак, – единственный корень.

Ответ : .

Пример 23 . Решить уравнение .

Решение . Опять-таки имеем стандартное иррациональное уравнение. Тем не менее, не будем спешить возводить в квадрат. Так, , , значит (функция возрастающая), и левая часть исходного уравнения не меньше 2. Следовательно, данное уравнение корней не имеет.

Ответ . Корней нет.

Пример 24 . Решить уравнение .

Решение . Поскольку и функция возрастающая, то . Следовательно, левая часть данного неравенства области определения принимает только отрицательные значения, то есть исходное уравнение корней не имеет.

Ответ : Корней нет.

Пример 25 . Решить уравнение .

Решение . Как и в предыдущих примерах, несложно обнаружить, что – корень. ОДЗ исходного уравнения – промежуток . Но теперь уже, в отличие от ранее рассмотренных задач, левая часть уравнения не задает монотонную функцию. Однако снова легко заметить, что на указанная функция возрастает, причем корень принадлежит этому промежутку. Значит, на данное уравнение имеет единственный корень. Осталось исследовать поведение функции на отрезке . Очевидно, что при , а . Следовательно, на исходное уравнение корней не имеет.

Ответ . .

Иногда знание ОДЗ позволяет доказать, что уравнение не имеет решений, а иногда позволяет найти решения уравнения непосредственной подстановкой чисел из ОДЗ.

Пример 26 . Решить уравнение .

Решение. ОДЗ этого уравнения состоит из всех , одновременно удовлетворяющих условиям и , то есть ОДЗ есть пустое множество. Этим решение уравнения завершается, так как установлено, что ни одно число не может являться решением, то есть уравнение не имеет корней.

Ответ : Корней нет.

Пример 27 . Решить уравнение .

Решение . Конечно, это иррациональное уравнение можно решить путем традиционного возведения обеих частей в квадрат. Однако, найдя ОДЗ этого уравнения, приходим к выводу, что ОДЗ исходного уравнения – одноэлементное множество <2>. Подставив в данное уравнение, приходим к выводу, что – корень исходного уравнения.

Ответ : .

3. Использование графиков функций

При решении уравнений или неравенств иногда полезно рассмотреть эскиз графиков их правой и левой частей в одной и той же системе координат. Тогда этот эскиз графиков поможет выяснить, на какие множества надо разбить числовую ось, чтобы на каждом из них решение уравнения (или неравенства) было очевидно.

Обратим внимание, что эскиз графика лишь помогает найти решение, но писать, что из графика следует ответ, нельзя, ответ еще надо обосновать.

Пример 28 . Решить уравнение .


Решение . ОДЗ данного уравнения есть все из промежутка . Эскизы графиков функций и представлены на рисунке 1.

Проведем прямую . Из рисунка следует, что график функции лежит не ниже этой прямой, а график функции не выше. При этом эти графики касаются прямой в разных точках. Следовательно, уравнение не имеет решений. Докажем это. Для каждого имеем , а . При этом только для , а только для . Это означает, что исходное уравнение не имеет корней.

Ответ : Корней нет.

Пример 29 . Решить уравнение .

Решение . Эскизы графиков функций и представлены на рисунке 2.


Легко проверяется, что точка является точкой пересечения графиков функций и , то есть – решение уравнения. Проведем прямую . Из рисунка следует, что она расположена между графиками функций и . Это наблюдение и помогает доказать, что других решений данное уравнение не имеет.

Для этого докажем, что для из промежутка справедливы неравенства и , а для промежутка справедливы неравенства и . Очевидно, что неравенство справедливо для , а неравенство для . Решим неравенство . Это неравенство равносильно неравенству , которое можно переписать в виде . Решениями этого неравенства являются все . Точно также показывается, что решениями неравенства являются все .

Следовательно, требуемое утверждение доказано, и исходное уравнение имеет единственный корень .

Ответ : .

Кроме рассмотренных типов иррациональных уравнений существуют еще и уравнения смешанного типа . К этой группе относятся иррациональные уравнения, содержащие кроме знака радикала и другие выражения (логарифмическое, показательное, тригонометрическое), а также знак модуля и параметр. Уравнения данного типа также чаще всего включаются в задания ЕГЭ и программу вступительных экзаменов в ВУЗы.

Со всеми учащимися на уроке такие уравнения разбирать не нужно, но они могут быть рассмотрены в рамках факультативных или кружковых занятий по математике с учащимися, повышенный интерес к математике. Примеры решения уравнений смешанного типа помещены в приложении А.

3. Тождественные преобразования при решении иррациональных уравнений

При решении иррациональных уравнений и неравенств часто приходится применять тождественные преобразования, связанные с использованием известных формул. К сожалению, эти действия иногда столь же небезопасны, как уже рассмотренное возведение в четную степень, – могут приобретаться или теряться решения. [17]

Рассмотрим несколько ситуаций, в которых эти проблемы наступают, и научимся их распознать и предотвращать.

I . Пример 30 . Решить уравнение .

Решение. При первом же взгляде на это уравнение возникает мысль избавиться от корня с помощью «преобразования» . Но это неверно, так как при отрицательных значениях x оказывалось бы, что . Здесь необходимо применить формулу . Уравнение теперь легко решается

.

Ответ . .

Рассмотрим «обратное» преобразование.

Пример 31 . Решить уравнение .

Решение. Здесь применима формула

.

Только необходимо задуматься о безопасности ее применения. Нетрудно видеть, что ее левая и правая части имеют разные области определения и что это равенство верно лишь при условии . Поэтому исходное уравнение равносильно системе

Решая уравнение этой системы, получим корни и . Второй корень не удовлетворяет совокупности неравенств системы и, следовательно, является посторонним корнем исходного уравнения.

Ответ. .

II . Следующее опасное преобразование при решении иррациональных уравнений, определяется формулой

.

Если пользоваться этой формулой слева направо, расширяется ОДЗ и можно приобрести посторонние решения. Действительно, в левой части обе функции и должны быть неотрицательны; а в правой неотрицательным должно быть их произведение. [17]

Пример 32 . Решить уравнение .

Решение . Возведем обе части уравнения в квадрат и произведем приведение подобных членов, перенос слагаемых из одной части равенства в другую и умножение обеих частей на . В результате получим уравнение

,

являющееся следствием исходного. Снова возведем обе части уравнения в квадрат. Получим уравнение

,

которое приводится к виду

.

Это уравнение (также являющееся следствием исходного) имеет корни , . Оба корня, как показывает проверка, удовлетворяют исходному уравнению.

Ответ . , .

Замечание. При возведении уравнения в квадрат учащиеся нередко в уравнении типа из Примера 32 производят перемножение подкоренных выражений, то есть вместо такого уравнения пишут уравнение

.

Такое «склеивание» не приводит к ошибкам, поскольку такое уравнение является следствием уравнения . Следует, однако, иметь в виду, что в общем случае такое перемножение подкоренных выражений дает неравносильные уравнения. Поэтому в рассмотренном выше примере можно было сначала перенести один из радикалов в правую часть уравнения, то есть уединить один радикал. Тогда в левой части уравнения останется один радикал, и после возведения обеих частей уравнения в квадрат в левой части уравнения получится рациональное выражение. [3]

Рассмотрим пример, где реализуется проблема с использованием формулы .

Пример 33 . Решить уравнение .

Решение. Попробуем решить это уравнение разложением на множители

.

Заметим, что при этом действии оказалось потерянным решение , так как оно подходит к исходному уравнению и уже не подходит к полученному: не имеет смысла при . Поэтому это уравнение лучше решать обычным возведением в квадрат

Решая уравнение этой системы, получим корни и . Оба корня удовлетворяют неравенству системы

Ответ. , .

Вывод. Есть два пути. Или аккуратно возводить уравнение в квадрат, или безошибочно определять, какие решения могли быть потеряны, и проверить, не случилось ли этого на самом деле.

III . Существует еще более опасное действие – сокращение на общий множитель. [17]

Пример 34 . Решить уравнение .

Неверное рассуждение: Сократим обе части уравнения на , получим

.

Нет ничего более опасного и неправильного, чем это действие. Во-первых, подходящее решение исходного уравнения было потеряно; во-вторых, было приобретено два посторонних решения . Получается, что новое уравнение не имеет ничего общего с исходным! Приведем правильное решение.

Решение . Перенесем все члены в левую часть уравнения и разложим ее на множители

.

Это уравнение равносильно системе

которая имеет единственное решение .

Ответ. .

Иррациональные неравенства – довольно сложный раздел школьного курса математики, а если учесть, что на его изучение отведено крайне мало времени, то становится ясно, что учащиеся как правило это раздел не усваивают. Даже у тех учащихся, что успешно решают иррациональные уравнения, часто возникают проблемы при решении иррациональных неравенств. Решение иррациональных неравенств осложняется тем обстоятельством, что здесь, как правило, исключена возможность проверки, поэтому надо стараться делать все преобразования равносильными.

3.1. Теоретические основы решения иррациональных неравенств

Если в любом иррациональном уравнении заменить знак равенства на один из знаков неравенства: >, , а класса школы №37 города Кирова. Ее основная задача: определить уровень подготовки, знаний и умений по теме «Иррациональные уравнения».

Учащимся было предложено 8 заданий, которые было необходимо выполнить в течение 1 часа. В классе 25 человек. Содержание диагностирующей контрольной работы №1 представлено в приложении Б.

Задания 1-3 –с выбором ответа, задания 4-7 – с кратким ответом, задание 8 – с развернутым ответом.

Результаты диагностирующей контрольной работы №1 отображены в таблице №1:

Кол-во человек, решивших задание

Доля человек, решивших задание в процентах

Проведение разработанной программы факультативных занятий.

Разработанные задания проводились 2 раза в неделю. Всего было проведено 6 занятий по 2 часа.

Основные задачи проведения факультативных занятий:

1) проверить правильность отбора содержания и системы упражнений;

2) выявить тот материал, который вызывает у учащихся наибольшие затруднения;

3) определить эффективность усвоения материала посредством текущей проверки;

4) выявить заинтересованность учащихся в изучении данной темы.

Проведение диагностирующей контрольной работы №2.

Контрольная работа была проведена после проведения факультативных занятий разработанной программы. Задача: выявление знаний и умений решать иррациональные уравнения.

Учащимся было предложено 8 заданий, которые было необходимо выполнить в течении 1 часа. Содержание диагностирующей контрольной работы №1 представлено в приложении Б.

Тематика заданий та же, что и в контрольной работе №1.

Результаты диагностирующей контрольной работы №2 отображены в таблице №2:

Кол-во человек, решивших задание

Доля человек, решивших задание в процентах

Анализ полученных результатов опытной работы.

Название: Методика решения иррациональных уравнений и неравенств в школьном курсе математики
Раздел: Рефераты по педагогике
Тип: дипломная работа Добавлен 08:28:58 26 мая 2008 Похожие работы
Просмотров: 3600 Комментариев: 21 Оценило: 4 человек Средний балл: 5 Оценка: неизвестно Скачать

На основании таблиц №1 и №2 можно построить диаграмму, отображающую сравнение результатов контрольных работ, проведенных перед посещением учащимися факультативных занятий и после их посещения.

Как видно из диаграммы, перед проведением факультативных занятий уровень знаний учащихся был средним, а после проведения занятий он повысился. Положительная тенденция заметна: учащиеся научились решать простейшие иррациональные уравнения и справились с заданиями 1-3, значительно лучше стало умение решать более сложные уравнения. Так как 8-ое задание относится к высокому уровню сложности, с ним справилось лишь 3 человека. Учащиеся лучше стали владеть методом введения новых переменных при решении иррациональных уравнений. Трудным показался материал, связанный с рационализирующими подстановками при решении иррациональных уравнений.

Программа факультативных занятий на тему «Иррациональные уравнения и методы их решения»

Ниже предлагается программа факультативных занятий на тему «Иррациональные уравнения и методы их решения». Курс лучше изучать в 11 классе, так как уравнения такого вида содержатся в заданиях ЕГЭ и на вступительных экзаменах в ВУЗы. Программа рассчитана на 16 часов. Занятия проводятся по 2 часа.

Тема: Равносильные и неравносильные преобразования уравнений.

1) Познакомить учащихся с понятием равносильных уравнений.

2) Показать, когда одно уравнение является следствием другого.

3) Сформулировать теоремы о равносильности уравнений.

4) Познакомить учащихся с равносильными и неравносильными преобразованиями уравнений.

Краткое содержание: Определение равносильности уравнений, следствия уравнений, понятие постороннего корня уравнения, перечисление и демонстрация на примерах равносильных и неравносильных преобразований уравнений.

Литература для учителя:

Литература для ученика:

Тема: Решение простейших иррациональных уравнений

1) Отработать у учащихся умение решать простейшие иррациональные уравнения вида , .

2) Закрепить изученный ранее материал.

3) Подготовить учащихся к изучению нового материала.

Краткое содержание: Определение иррационального уравнения, решение простейших иррациональных уравнений вида , методом возведения обеих частей уравнения в одну и ту же степень с последующей проверкой полученных корней, а также методом сведения к равносильной системе уравнений и неравенств. Метод уединения радикала.

Литература для учителя:

Литература для ученика:

Тема: Решение иррациональных уравнений методом замены.

Цель: Научить учащихся решать иррациональные уравнения методом замены.

Краткое содержание: Применение метода замены в случае, если в уравнении неоднократно встречается некоторое выражение. Решение иррациональных уравнений методом сведения к эквивалентным системам рациональных уравнений при помощи введения двух вспомогательных неизвестных.

Литература для учителя:

Литература для ученика:

Тема: Применение рационализирующих подстановок при решении иррациональных уравнений.

Цель: Научить учащихся решать иррациональные уравнения при помощи рационализирующих подстановок.

Краткое содержание: Рассмотрение рационализации некоторых выражений, содержащих радикалы, с помощью рационализирующих подстановок и применение этих подстановок при решении иррациональных уравнений.

Литература для учителя:

Литература для ученика:

Тема: Решение иррациональных уравнений функционально-графическим методом.

Цель: Научить учащихся решать иррациональные уравнения и неравенства, используя свойства входящих в них функций.

Краткое содержание: Использование ОДЗ, монотонности, графиков функций при решении иррациональных уравнений.

Литература для учителя:

Литература для ученика:

Тема: Обобщение и систематизация методов решения иррациональных уравнений.

1) Показать учащимся, что иррациональные уравнения можно решать не одним методом.

2) Систематизировать методы решения иррациональных уравнений.

3) Научить выбирать наиболее рациональный способ решения.

Краткое содержание: Рассмотрение различных методов решения на примере одного иррационального уравнения вида .

Литература для учителя:

Литература для ученика:

Тема: Иррациональные уравнения, содержащие знак модуля или параметр. Решение уравнений смешанного типа.

Цель: Показать учащимся как решаются уравнения смешанного типа и уравнения, содержащие знак модуля и параметр.

Краткое содержание: Решение иррациональных уравнений с параметром и модулем, а также иррациональные уравнения, содержащие логарифмические, показательные или тригонометрические выражения.

Литература для учителя:

Литература для ученика:

В данной работе сделана попытка разработать методику обучения решению иррациональных уравнений и неравенств в школе.

При проведении исследования были решены следующие задачи:

1) Проанализированы действующие учебники алгебры и начала математического анализа для выявления представленной в них методики решения иррациональных уравнений и неравенств. Проведенный анализ позволяет сделать следующие выводы:

·в средней школе недостаточное внимание уделяется методам решения различных иррациональных уравнений, в основном программой предусмотрено формирование у учащихся решать простейшие иррациональные уравнения и неравенства;

·в учебнике [1] материала, посвященного методам решения иррациональных уравнений нет. В остальных учебниках рассмотрены два основных способа решения: возведение обеих частей уравнения в степень, с последующей подстановкой полученных корней в исходное уравнение, а также решение уравнений с помощью равносильных преобразований;

·очень мало материала по методам решения иррациональных неравенств;

·среди предлагаемых заданий в учебниках много однотипных;

2) Изучена учебно-методическая литература по данной теме;

3) Рассмотрены основные методы и приемы решения различных иррациональных уравнений и неравенств;

4) Рассмотрены ситуации, связанные с потерей или приобретением посторонних корней в процессе решения, показано, как распознавать и предотвращать их;

5) Подобраны примеры решения иррациональных уравнений и неравенств для демонстрации излагаемого теоретического материала;

1. Алимов Ш. А. Алгебра и начала анализа [Текст]: учебник для 10-11 класса средней школы / Ш. А. Алимов – М.: Просвещение, 1993. – 254 с.

2. Башмаков М. И. Алгебра и начала анализа [Текст]: учебник для 10-11 класса средней школы / М. И. Башмаков – М.: Просвещение, 1992. – 351 с.

3. Болтянский В. Г. Математика: лекции, задачи, решения [Текст] / В. Г. Болтянский – Литва: Альфа, 1996. – 637 с.

4. Виленкин Н. Я. и др. Алгебра и математический анализ для 11 класса [Текст]: учебное пособие для учащихся школ и классов с углубленным изучением математики / Н. Я. Виленкин – М.: Просвещение, 1998. – 288 с.

5. Галицкий М. Л. Сборник задач по алгебре для 8-9 классов [Текст]: учебное пособие для учащихся школ и классов с углубленным изучением математики М. Л. Галицкий – М.: Просвещение, 1999. – 271с.

6. Григорьев А. М. Иррациональные уравнения [Текст] / А. М. Григорьев // Квант. – 1972. – №1. – С. 46-49.

7. Денищева Л. О. Готовимся к единому государственному экзамену. Математика. [Текст] / Л. О. Денищева – М.: Дрофа, 2004. – 120 с.

8. Егоров А. Иррациональные неравенства [Текст] / А Егоров // Математика. Первое сентября. – 2002. – №15. – С. 13-14.

9. Егоров А. Иррациональные уравнения [Текст] / А Егоров // Математика. Первое сентября – 2002. – №5. – С. 9-13.

10. Мордкович А. Г. Алгебра и начала анализа. 10-11 класс [Текст]: В двух частях. Ч.1: учебник для общеобразовательных учреждений / А. Г. Мордкович – М.: Мнемозина, 2004. – 315 с.

11. Мордкович А. Г. Алгебра и начала анализа. 10-11 класс [Текст]: В двух частях. Ч.2: задачник для общеобразовательных учреждений / А. Г. Мордкович – М.: Мнемозина, 2004. – 315 с.

12. Мордкович А. Г. Кто-то теряет, кто-то находит [Текст] / А. Г. Мордкович // Квант – 1970. – №5. – С. 48-51.

13. Колмогоров А. Н. Алгебра и начала анализа [Текст]: учебник для 10-11 класса средней школы / А. Н. Колмогоров – М.: Просвещение, 1991. – 320 с.

14. Кузнецова Г. М. Программа для общеобразовательных школ, гимназий, лицеев: Математика. 5-11 классы [Текст] / Г. М. Кузнецова – М.: Дрофа, 2004 – 320 с.

15. Потапов М. Как решать уравнения без ОДЗ [Текст] / М. Потапов // Математика. Первое сентября – 2003. – №21. – С. 42-43.

16. Соболь Б. В. Пособие для подготовки к единому государственному экзамену и централизованному тестированию по математике [Текст] / Б. В. Соболь – Ростов на Дону: Феникс, 2003. – 352 с.

17. Черкасов О. Ю. Математика [Текст]: справочник для старшеклассников и поступающих в вузы / О. Ю. Черкасов – М.: АСТ-ПРЕСС, 2001. – 576 с.

18. Шабунин М. Лекции для абитуриентов. Лекция 1. [Текст] / М. Шабунин // Математика. Первое сентября – 1996. – №24. – С. 24.

19. Шувалова Э. З. Повторим математику [Текст]: учебное пособие для поступающих в вузы / Э. З. Шувалова – М.: Высшая школа, 1974. – 519 с.

20. Моденов В. П. Решение иррациональных уравнений [Текст] / В. П. Моденов // Математика в школе – 1970. – №6. – С. 32-35.

21. Горнштейн П. И. Экзамен по математике и его подводные рифы [Текст] / П. И. Горнштейн – М.: Илекса, Харьков: Гимназия, 1998, – 236 с.

24. Шарова Л. И. Уравнения и неравенства [Текст]: пособие для подготовительных отделений / Л. И. Шарова – Киев: Вища школа, 1981. – 280 с.

26. Егоров А. Иррациональные неравенства [Текст] / А Егоров // Математика. Первое сентября. – 2002. – №17. – С. 13-14.

27. Мордкович А. Г. Алгебра. 8 класс [Текст]: В двух частях. Ч.1: учебник для общеобразовательных учреждений / А. Г. Мордкович – М.: Мнемозина, 2004. – 315 с.

28. Мордкович А. Г. Алгебра. 8 класс [Текст]: В двух частях. Ч.2: задачник для общеобразовательных учреждений / А. Г. Мордкович – М.: Мнемозина, 2003. – 239 с.

Решение иррациональных уравнений смешанного типа

Для каждого вида уравнений и неравенств, в том числе и иррациональных, можно составить уравнение или неравенство «с модулем» и «с параметром» .

Иррациональные уравнения, содержащие знак модуля

Простейшие уравнения с модулем имеют вид: и ; будем их решать на основании определения модуля сведением к совокупности систем.

Пример 1 . Решить уравнение .

Решение . ,

Данное уравнение равносильно совокупности двух систем:

Будем решать каждую из систем по отдельности.

Решение первой системы:

Последняя система не имеет корней, так как дискриминант уравнения меньше нуля.

Решение второй системы:

Ответ : .

Пример 2 . Решить уравнение

Решение . ,

Данное уравнение равносильно совокупности двух систем:

Будем решать каждую из систем по отдельности.

Решение первой системы:

Если внимательно посмотреть на неравенства последней системы, можно заметить, что пересечение множеств и пусто. Следовательно, первая система совокупности корней не имеет.

Решение второй системы:

Ответ : .

Иррациональные уравнения, содержащие параметр

Уравнение вида называется иррациональным с параметром относительно неизвестного , если одна или обе его части содержат выражения, иррациональные относительно .

Как и раньше, будем находить только действительные корни.

Трудно указать какой-нибудь общий и вместе с тем достаточно простой способ решения иррациональных уравнений, содержащих параметр.

Проиллюстрируем некоторые способы решения на примерах.

Пример 3 . Для каждого действительного значения параметра решить уравнение

.

Решение . Исходное уравнение равносильно смешанной системе

При эта система решений не имеет.

При получим решение

Теперь необходимо найти те значения , при которых эта система имеет решение:

Ответ : при – корней нет;

при .

Для решения иррационального уравнения иногда удобно ввести вспомогательную неизвестную величину. При этом получаем квадратное уравнение с параметром, которое нужно решить в пределах некоторого ограниченного множества значений нового неизвестного.

Пример 4 . Решить уравнение .

Решение . Область определения данного уравнения:

Так как и , то и .

Сделаем замену , тогда и исходное уравнение можно записать в виде системы

которая равносильна системе

Корни уравнения должны удовлетворять первому условию последней системы, то есть необходимо решить систему

Итак, при исходное уравнение имеет единственный корень . Отсюда при имеем

,

Ответ : при ;

при – корней нет.

Иррациональные показательные уравнения

Пример 5 . Решить уравнение .

Решение . Перепишем уравнение так:

,

Приведем все степени к одному основанию 7:

.

Сделаем замену , , тогда получаем уравнение , корнями которого являются

Сделаем обратную замену:

или

– уравнение не имеет решений.

Ответ : .

Пример 6 . Решить уравнение .

Решение . Приведем все степени к одному основанию:

.

откуда получаем уравнение которое равносильно уравнению:

Ответ :

Иррациональные логарифмические уравнения

Пример 7 . Решить уравнение .

Решение . Преобразуем данное уравнение:

.

Учитывая ОДЗ, данное уравнение равносильно системе:

Ответ :

Пример 8. Решить уравнение

Решение . Учитывая ОДЗ, данное уравнение равносильно системе:

Уравнение этой системы равносильно совокупности уравнений:

Последнее уравнение этой совокупности равносильно уравнению:

Из неравенства системы следует, что . Следовательно, – посторонний корень.

Ответ : ,

Сколько корней имеет уравнение ?

Сколько корней имеет уравнение ?

Диагностирующая контрольная работа №1

1. Сколько корней имеет уравнение ?

2. Решите уравнение, укажите корень уравнения (или сумма корней, если их несколько).

А.

3. Укажите промежуток, которому принадлежит корень уравнения (или сумма корней, если их несколько).

А.

Б.

В.

Г.

4. Решите уравнение, укажите корень уравнения (или произведение корней, если их несколько).

5. Решите уравнение , укажите корень уравнения.

6. Решите уравнение , укажите корень уравнения (если корень не единственный, то наибольший)

7. Решите уравнение , укажите корень уравнения.

8. Решите уравнение .

Диагностирующая контрольная работа № 2

1. Сколько корней имеет уравнение ?

2. Решите уравнение, укажите корень уравнения (или сумма корней, если их несколько).

В.

3. Укажите промежуток, которому принадлежит корень уравнения (или сумма корней, если их несколько).

А.

Б.

В.

Г.

4. Решите уравнение, укажите корень уравнения (или произведение корней, если их несколько).

5. Решите уравнение , укажите корень уравнения.

6. Решите уравнение , укажите корень уравнения (если корень не единственный, то наибольший).

7. Решите уравнение , укажите корень уравнения.

8. Решите уравнение .

Ответы и решение заданий диагностирующей контрольной работы №1

4. Уединив первый радикал, получаем уравнение , равносильное исходному. Возводя обе части этого уравнения в квадрат, получаем уравнение, . Последнее уравнение равносильно системе Решая уравнение этой системы, равносильное уравнению , получим корни и . Первый корень не удовлетворяет неравенству системы и, следовательно, является посторонним корнем исходного уравнения. Ответ: .

5. Введем новую переменную , тогда , причем . В результате исходное иррациональное уравнение принимает вид квадратного , откуда учитывая ограничение , получаем . Решая уравнение , получаем корень . Как показывает проверка, удовлетворяет исходному уравнению. Ответ: .

6. Введем новую переменную . В результате исходное иррациональное уравнение принимает вид Решая первое уравнение этой системы, получим корни и . Второй корень не удовлетворяет неравенству системы. Решая уравнение , получаем корни и . Как показывает проверка, оба корня удовлетворяют исходному уравнению. В ответе нужно указать наибольший из корней. Ответ: .

7. Данное уравнение равносильно совокупности двух систем: и Будем решать каждую из систем по отдельности. Решение первой системы: Если внимательно посмотреть на неравенства последней системы, можно заметить, что пересечение множеств и пусто. Следовательно, первая система совокупности корней не имеет. Решение второй системы: Решая уравнение этой системы, равносильное уравнению , получим корни и . Второй корень не удовлетворяет неравенству системы и, следовательно, является посторонним корнем исходного уравнения. Ответ: .

8. Введем новые переменные и . Тогда исходное уравнение принимает вид: . Поскольку мы ввели две новые неизвестные, надо найти еще одно уравнение, связывающее y и z . Для этого возведем равенства , в третью степень и заметим, что . Итак, надо решить систему уравнений она имеет два (действительных) решения: , ; , . Остается решить систему двух уравнений с одним неизвестным и систему первая из них дает , вторая дает . Как показывает проверка, оба корня удовлетворяют исходному уравнению. Ответ: , .

Ответы и решение заданий диагностирующей контрольной работы №2

4. Уединив первый радикал, получаем уравнение , равносильное исходному. Возводя обе части этого уравнения в квадрат, получаем уравнение,. Последнее уравнение равносильно системе Решая уравнение этой системы, равносильное уравнению , получим корни и . Оба корня удовлетворяют неравенству системы и, следовательно, являются корнями исходного уравнения. В ответе нужно указать произведение корней. Ответ: 48.

5. Введем новую переменную , тогда , причем . В результате исходное иррациональное уравнение принимает вид квадратного , откуда учитывая ограничение , получаем . Решая уравнение , получаем корень . Как показывает проверка, удовлетворяет исходному уравнению. Ответ: .

6. Введем новую переменную . В результате исходное иррациональное уравнение принимает вид Решая первое уравнение этой системы, равносильное уравнению , получим корни и . Первый корень не удовлетворяет неравенству системы. Решая уравнение , получаем корни и . Как показывает проверка, оба корня удовлетворяют исходному уравнению. В ответе нужно указать наибольший из корней. Ответ: .

7. Данное уравнение равносильно совокупности двух систем: и Будем решать каждую из систем по отдельности. Решение первой системы: Решая уравнение этой системы, равносильное уравнению , получим корни и . Второй корень не удовлетворяет неравенству системы и, следовательно, является посторонним корнем исходного уравнения. Решение второй системы: Решая уравнение этой системы, равносильное уравнению , получим корни и . Оба корня не удовлетворяют неравенству системы и, следовательно, являются посторонними корнями исходного уравнения. Ответ: .

8. Введем новые переменные и . Тогда исходное уравнение принимает вид: . Поскольку мы ввели две новые неизвестные, надо найти еще одно уравнение, связывающее y и z . Для этого возведем равенства , в четвертую степень и заметим, что . Итак, надо решить систему уравнений она имеет два (действительных) решения: , ; , . Остается решить систему двух уравнений с одним неизвестным и систему первая из них дает , вторая дает . Как показывает проверка, оба корня удовлетворяют исходному уравнению. Ответ: , .

Разработка факультативного занятия на тему «Способ рационализации при решении иррациональных уравнений»

Иногда посредством некоторой подстановки удается привести иррациональное уравнение к рациональному виду. В таком случае говорят, что эта подстановка рационализирует рассматриваемое иррациональное уравнение, и называют ее рационализирующей.

Способ решения иррациональных уравнений, основанный на применении рационализирующих подстановок, назовем способом рационализации .

Применяя рационализирующую подстановку, необходимо следить за тем, чтобы область определения нового рационального уравнения, получаемого в результате этой подстановки, соответствовала области определения данного иррационального уравнения. Только при этом условии рационализирующая подстановка приведет рассматриваемое иррациональное уравнение к рациональному уравнению, которое всюду в области его определения эквивалентно данному.

Рассмотрим рационализацию некоторых выражений, содержащих радикалы, с помощью рационализирующих подстановок и применение этих подстановок при решении иррациональных уравнений.

1. Рационализация выражения

, (1)

где обозначает рациональную функцию, и – постоянные, а – любое целое положительное число, рационализируется подстановкой

. (2)

Действительно, возводя обе части равенства (2) в -ую степень, получим , откуда , причем функция рациональна. Следовательно,

.

Поскольку рациональная функция от рациональной функции представляет собой также рациональную функцию, то выражение, стоящее в правой части последнего равенства, является рациональным.

Пример 1 . Решить уравнение .

Решение . ОДЗ рассматриваемого уравнения . Рационализирующей подстановкой это уравнение приводится к эквивалентной ему смешанной системе

или (сокращая дробь на ) системе

Решением последней будет . Воспользовавшись подстановкой, получим .

Ответ : .

2. Рациональность дробно-линейных иррациональностей

Аналогично предыдущему доказывается, что функция вида

, (3)

где , , и – некоторые постоянные, а – любое целое положительное число (дробно-линейная иррациональность), может быть при условии приведена к рациональному виду подстановкой

(4)

(5)

рационализируется при помощи подстановки

(6)

где – наименьшее общее кратное показателей радикалов , , …

Пример 2 . Решить уравнение .

Решение . Будем искать корни данного уравнения в области (очевидно, что числа и не являются его корнями). Разделим обе части уравнения на :

.

Полученное уравнение в рассматриваемой области с помощью рационализирующей подстановки

сводится к смешанной системе

эквивалентной ему в этой области. Определив решения этой системы и и воспользовавшись подстановкой, находим корни исходного уравнения.

Ответ : .

3. Рационализация биноминальных выражений

Можно доказать, что выражение

, (7)

где и – постоянные, а показатели степеней , – некоторые рациональные числа, допускает рационализирующие подстановки только в трех случаях , когда оказывается целым одно из чисел , или .

В этих случаях возможны следующие подстановки:

Если – целое, то , где – наименьшее общее кратное знаменателей чисел и .

Если – целое, то , где – знаменатель числа .

Если – целое, то , где – знаменатель числа .

Существование указанных трех рационализирующих подстановок доказывает возможность приведения к рациональному виду уравнений в первом случае и во втором и третьем случаях.

Пример 3 . Решить уравнение .

Решение . Так как – не является корнем уравнения, разделим обе его части на . Выделяется биномиальное выражение:

.

Имеет место третий случай рационализации ( и – целое число). Следовательно, будем применять подстановку . Возводя обе части этого равенства в квадрат, получим , так что . Теперь с помощью подстановки и найденного значения получаем

и исходное иррациональное уравнение приводится к рациональному , или . Определив корни этого уравнения , и воспользовавшись подстановкой, находим

Ответ :

4. Рационализация квадратичных иррациональностей посредством подстановок Эйлера

Квадратичной иррациональностью назовем функцию вида

, (9)

где и – некоторые постоянные. Покажем, что это выражение всегда рационализируется одной из так называемых подстановок Эйлера. При этом мы, конечно, будем считать, что квадратный трёхчлен неотрицателен и не имеет равных корней (в противном случае корень можно заменить рациональным выражением).

а) Сначала рассмотрим случай, когда дискриминант . В этом случае знак квадратного трёхчлена совпадает со знаком , и поскольку этот трёхчлен положителен (в силу условия равенство трёхчлена нулю невозможно), то .

Таким образом, мы можем сделать следующую подстановку:

(или ) (10)

Подстановку (10) иногда называют первой подстановкой Эйлера . Докажем, что эта подстановка рационализирует функцию (9) в рассматриваемом случае. Возводя в квадрат обе части равенства

(заметим, что ), получим , так что

,

где функции и рациональные. Таким образом,

.

В правой части полученного равенства стоит рациональная функция.

б) Рассмотрим теперь случай, когда дискриминант , то есть квадратный трехчлен имеет (различные) действительные корни и . Следовательно,

.

Аналогично предыдущему доказывается, что в этом случае функция (9) рационализируется посредством подстановки:

, (11)

называемой часто второй подстановкой Эйлера .

Замечание 1. Рационализирующая подстановка (11) справедлива при условии . Следовательно, применяя эту подстановку при решении иррационального уравнения, необходимо проверить, не является ли значение корнем данного уравнения (иначе возможна потеря этого корня).

Замечание 2. Если , то в этом случае можно положить

(или ) (12)

Ответ : , .

Пример 4 . Решить уравнение .

Решение . В данном уравнении дискриминант квадратного трехчлена положителен, корни его и . Найдем другие корни подстановкой

.

Применяя эту подстановку, необходимо проверить, не является ли значение корнем данного уравнения. Итак, – корень данного уравнения.

Возводя в квадрат обе части равенства , получим , откуда . Теперь подставим это значение в исходное уравнение и последовательно получаем:

и исходное уравнение сводится к уравнению , или . Это уравнение имеет единственный действительный корень , тогда . Итак, исходное уравнение имеет два корня: и .

Ответ : , .

5. Рационализация с помощью тригонометрических подстановок

Иногда подходящей заменой неизвестной иррациональное уравнение можно свести к тригонометрическому уравнению. При этом полезными могут оказаться следующие замены переменной. [17]

1). Если в уравнение входит радикал , то можно сделать замену , или , .

2). Если в уравнение входит радикал , то можно сделать замену tg t , или ctg t , .

3). Если в уравнение входит радикал , то можно сделать замену , или , .

Проиллюстрируем использование этих замен на следующих примерах.

Пример 5 . Решить уравнение .

Решение . В данное уравнение входит выражение , поэтому в соответствии с пунктом 2, сделаем замену

tg t , где .

Тогда выражение , входящее в уравнение, можно преобразовать

и исходное уравнение можно записать в виде

.

Поскольку не равен нулю при рассматриваемых значениях t , то полученное уравнение равносильно уравнению

.

Решая это уравнение, находим два возможных значения

и .

Из всех корней этих уравнений промежутку принадлежит единственное значение .

Поэтому соответствующее значение x равно

.

Ответ . .

Пример 6. Решить уравнение .

Решение . В этом уравнении x по ОДЗ может принимать только значения из отрезка , что приводит к мысли совершить замену

, где .

В результате такой замены приходим к уравнению

.

и ,

.

В силу ограничения выполнено , поэтому приходим к уравнению

,

которое, пользуясь формулой приведения, сведем к стандартному виду

.

Решая последнее уравнение, находим

или , .

Условию удовлетворяют лишь три значения

, , .

, , .

Ответ . , , .

В заключение нужно отметить, что способ рационализации успешно может быть применён также для рационализации иррациональных неравенств, для вычисления и преобразования иррациональных выражений и так далее.

Алгебра и начала математического анализа. 10 класс

Конспект урока

Алгебра и начала математического анализа, 10 класс

Урок №20. Иррациональные уравнения и неравенства

Перечень вопросов, рассматриваемых в теме

1) понятие иррационального уравнения;

2) понятие иррационального неравенства;

3) виды и методы решения простейших иррациональных уравнений;

4) методы решения иррациональных неравенств.

Глоссарий по теме

Иррациональное уравнение – это уравнения, в которых неизвестное находится под знаком корня.

Свойство: при возведении обеих частей уравнения в натуральную степень получается уравнение – следствие данного.

Колягин Ю.М., Ткачева М.В, Федорова Н.Е. и др., под ред. Жижченко А.Б. Алгебра и начала математического анализа (базовый и профильный уровни) 10 кл. – М.: Просвещение, 2014.

Шабунин М.И., Ткачева М.В., Федорова Н.Е. Дидактические материалы Алгебра и начала математического анализа (базовый и профильный уровни) 10 кл. – М.: Просвещение, 2017.

Теоретический материал для самостоятельного изучения

Иррациональное уравнение – это уравнения, в которых неизвестное находится под знаком корня.

Свойство: при возведении обеих частей уравнения в натуральную степень получается уравнение – следствие данного.

Рассмотрим виды иррациональных уравнений

В этом случае мы можем воспользоваться определением квадратного корня.

Из него следует, что а≥0, тогда

Для нашего случая получим

или

Мы знаем, что сумма положительных чисел равна нулю тогда и только тогда, когда каждое из слагаемых равно нулю.
Т.е.

По определению квадратного корня f(x) > 0. Таким образом, чтобы найти такие значения неизвестной, при которых выполняются следующие условия:

следовательно, решений нет

Ответ: решений нет

Определение. Неравенство, содержащие переменную под знаком корня, называется иррациональным.

Иррациональное неравенство, как правило, сводится к равносильной системе (или совокупности систем) неравенств.

Разбор решения заданий тренировочного модуля

Решим уравнение:

Возведем в квадрат обе части уравнения, получим:

, которое не будет равносильно исходному уравнению, потому что у этого уравнения два корня , а у первоначального уравнения только один корень х=4.

Подчеркните корни данного уравнения

Решим данное уравнение.

Получаем три корня из последнего уравнения: -1;0;1

Решите уравнение:

Рассмотрим область определения функций:

х=-2, но -2 не входит в область определения функций, следовательно, решений нет.


источники:

http://www.bestreferat.ru/referat-188158.html

http://resh.edu.ru/subject/lesson/5569/conspect/